安振平老师的4958号不等式问题的证明

题目:已知$a,b,c\geq 0$,$a+b+c=3$,求证:$\frac{a}{a^2+b+c}+\frac{b}{b^2+c+a}+\frac{c}{c^2+a+b}\leq 1$.

证明:因为$a+b+c=3$,又$3(a^2+b^2+c^2)-(a+b+c)^2=(a-b)^2+(b-c)^2+(c-a)^2\geq 0$. 所以

$a^2+b^2+c^2\geq \frac{1}{3}(a+b+c)^2\geq a+b+c$.           (1)

由柯西不等式及不等式(1)

$\frac{a}{a^2+b+c}+\frac{b}{b^2+c+a}+\frac{c}{c^2+a+b}=\frac{a(1+b+c)}{(a^2+b+c)(1+b+c)}+\frac{b(1+c+a)}{(b^2+c+a)(1+c+a)}+\frac{c(1+a+b)}{(c^2+a+b)(1+a+b)}$

$\leq \frac{a(1+b+c)+b(1+c+a)+c(1+a+b)}{(a+b+c)^2}=\frac{a+b+c+2(ab+bc+ca)}{(a+b+c)^2}\leq\frac{a^2+b^2+c^2+2(ab+bc+ca)}{(a+b+c)^2}=1$.

故原不等式获证.

 

转载于:https://www.cnblogs.com/ydwu/p/10669715.html

评论
添加红包

请填写红包祝福语或标题

红包个数最小为10个

红包金额最低5元

当前余额3.43前往充值 >
需支付:10.00
成就一亿技术人!
领取后你会自动成为博主和红包主的粉丝 规则
hope_wisdom
发出的红包
实付
使用余额支付
点击重新获取
扫码支付
钱包余额 0

抵扣说明:

1.余额是钱包充值的虚拟货币,按照1:1的比例进行支付金额的抵扣。
2.余额无法直接购买下载,可以购买VIP、付费专栏及课程。

余额充值